Discrete Math- Irrational numbers, proof or counterexample

Click For Summary
The statement that if r is any rational number and s is any irrational number, then r/s is irrational, is false. A counterexample is provided with r = 0 and s = √2, resulting in r/s = 0, which is rational. The discussion reveals that some participants believe the statement is true and have attempted to provide proofs, but they have misunderstood the concept. The correct approach involves recognizing that a single counterexample suffices to disprove the statement. Understanding the definitions of rational and irrational numbers is crucial in resolving this problem.
abjf9299
Messages
5
Reaction score
0

Homework Statement



Determine if the statement is true or false. Prove those that are true and give a counterexample for those that are false.

If r is any rational number and if s is any irrational number, then r/s is irrational.

Homework Equations



A rational number is equal to the ratio of two other numbers.
An irrational number can't be expressed as the ratio of two other numbers.


The Attempt at a Solution



I said that this statement is false. As my counterexample, I set r = 0 and s = (2)^1/2 .

r/s then equals 0 which is rational.


I have seen several people give different answers to this problem (our professor let's us consult with each other on the homework). Am I right? If I am wrong, could someone give me a proof for this problem?
 
Physics news on Phys.org
abjf9299 said:
If r is any rational number and if s is any irrational number, then r/s is irrational.

You're correct. To prove this is false, you need to provide a counterexample for one situation, which you have done. By different answers though, what do you mean? Do some people think it's true, or are they providing different counterexamples? If they're just providing different counterexamples, there's nothing wrong with that.
 
Thanks for the answer! By different answers I mean they think it's true and provided "proofs" to support their assertions, but I know where they made their mistakes now. Thanks again for your help!
 
Question: A clock's minute hand has length 4 and its hour hand has length 3. What is the distance between the tips at the moment when it is increasing most rapidly?(Putnam Exam Question) Answer: Making assumption that both the hands moves at constant angular velocities, the answer is ## \sqrt{7} .## But don't you think this assumption is somewhat doubtful and wrong?

Similar threads

  • · Replies 8 ·
Replies
8
Views
2K
  • · Replies 2 ·
Replies
2
Views
3K
  • · Replies 4 ·
Replies
4
Views
5K
  • · Replies 8 ·
Replies
8
Views
2K
  • · Replies 15 ·
Replies
15
Views
5K
  • · Replies 3 ·
Replies
3
Views
9K
  • · Replies 25 ·
Replies
25
Views
3K
  • · Replies 7 ·
Replies
7
Views
3K
Replies
10
Views
2K
  • · Replies 2 ·
Replies
2
Views
2K